- PowerScore Staff
- Posts: 5972
- Joined: Mar 25, 2011
- Thu Jul 14, 2016 3:38 pm
#27193
Complete Question Explanation
(The complete setup for this game can be found here: lsat/viewtopic.php?t=11429)
The correct answer choice is (B)
Remember to apply the simplest and most visually powerful rules first. In this case that happens to be the order in which the rules are presented. The first rule eliminates answer choice (C). The second rule eliminates answer choice (D). The third rule eliminates answer choice (A). And the fourth rule eliminates answer choice (E). Answer choice (B) is thus correct.
(The complete setup for this game can be found here: lsat/viewtopic.php?t=11429)
The correct answer choice is (B)
Remember to apply the simplest and most visually powerful rules first. In this case that happens to be the order in which the rules are presented. The first rule eliminates answer choice (C). The second rule eliminates answer choice (D). The third rule eliminates answer choice (A). And the fourth rule eliminates answer choice (E). Answer choice (B) is thus correct.
Dave Killoran
PowerScore Test Preparation
Follow me on X/Twitter at http://twitter.com/DaveKilloran
My LSAT Articles: http://blog.powerscore.com/lsat/author/dave-killoran
PowerScore Podcast: http://www.powerscore.com/lsat/podcast/
PowerScore Test Preparation
Follow me on X/Twitter at http://twitter.com/DaveKilloran
My LSAT Articles: http://blog.powerscore.com/lsat/author/dave-killoran
PowerScore Podcast: http://www.powerscore.com/lsat/podcast/